Your-Doctor
Multiple Choice Questions (MCQ)



Free Palestine
Quiz Categories Click to expand

Category: Cardiology--->CT, CMR, And Nuclear Imaging
Page: 4

Question 16# Print Question

A 66-year-old man presents to the outpatient clinic with breathlessness on exertion. He is a smoker with treated hypertension. A TTE reveals a dilated LV with overall moderate LV systolic dysfunction. He has a CMR to try to elucidate the cause of the LV systolic dysfunction.

Video below shows the long-axis cines 

Video below shows the short-axis cines.

Figure below shows late myocardial enhancement following gadolinium in the four- and two-chamber views (top row) and at the basal, mid, and apical levels (bottom row, left to right).

What is the diagnosis?

A. Dilated cardiomyopathy
B. Anterior myocardial infarction
C. Arrhythmogenic cardiomyopathy
D. LV non-compaction
E. Hypertrophic cardiomyopathy


Question 17# Print Question

You are asked to arrange a cardiac MRI to assess the left ventricular function of a patient following incomplete revascularization by percutaneous coronary intervention and stent implantation. 

At what stage following the stent implantation is it safe to perform the scan?

A. Immediately—there is no time limit
B. Never—the static magnetic field will displace the bare metal stent
C. Three months, to allow for endothelization of the stent struts
D. After 4 weeks following cessation of clopidogrel
E. None of the above


Question 18# Print Question

Which one of the following is an absolute contraindication for an MRI scan?

A. An all-metal aortic valve replacement
B. A St Jude mitral valve replacement
C. A total hip replacement
D. A bare metal stent in the LMS
E. A cerebral aneurysm clip of unknown source


Question 19# Print Question

A 60-year-old man presents with angina and heart failure. His estimated ejection fraction by echocardiography is 25%. An invasive coronary angiogram demonstrates widespread severe three-vessel coronary disease with good distal targets. A CMR study shows an ejection fraction of 22% and <25% myocardial wall thickness of hyper-enhancement in the mid and apical inferior segments.

Which one of the following statements is correct?

A. The chance of functional recovery in the LAD territory is <20%
B. The patient should not be offered revascularization because of the poor chance of functional and prognostic improvement
C. The RCA territory has a >60% chance of functional recovery if revascularized
D. He should be offered PCI to the RCA only
E. His prognosis is better if he is treated medically than if he is completely revascularized


Question 20# Print Question

A 55-year-old man presents with a 2-week history of dyspnoea following an episode of severe chest pain. An invasive coronary angiogram shows a 95% stenosis in the proximal LAD and an akinetic anterior wall. He is referred for a cardiac MRI viability study prior to percutaneous revascularization.

Which one of the following statements is correct?

A. He should have a stent implanted anyway without the MRI
B. 100% hyper-enhancement suggests that he should go forward for PCI to the LAD
C. The scan should be done following the PCI
D. Severe hypokinesis of the anterior wall suggest that the LAD territory is non-viable
E. If a transmural infarct (100% enhancement) is present he should not have a PCI but should have medical therapy




Category: Cardiology--->CT, CMR, And Nuclear Imaging
Page: 4 of 10